Difference between revisions of "2020 AMC 8 Problems/Problem 2"

(Solution2)
m (Solution1: Since there is only one solution, labeling "Sol 1" is unnecessary.)
Line 4: Line 4:
 
<math>\textbf{(A) }\$5 \qquad \textbf{(B) }\$10 \qquad \textbf{(C) }\$15 \qquad \textbf{(D) }\$20 \qquad \textbf{(E) }\$25</math>
 
<math>\textbf{(A) }\$5 \qquad \textbf{(B) }\$10 \qquad \textbf{(C) }\$15 \qquad \textbf{(D) }\$20 \qquad \textbf{(E) }\$25</math>
  
==Solution1==
+
==Solution==
 
The friends earn <math>\$\left(15+20+25+40\right)=\$100</math> in total. Since they decided to split their earnings equally, it follows that each person will get <math>\$\left(\frac{100}{4}\right)=\$25</math>. Since the friend who earned <math>\$40</math> will need to leave with <math>\$25</math>, he will have to give <math>\$\left(40-25\right)=\boxed{\textbf{(C) }\$15}</math> to the others.
 
The friends earn <math>\$\left(15+20+25+40\right)=\$100</math> in total. Since they decided to split their earnings equally, it follows that each person will get <math>\$\left(\frac{100}{4}\right)=\$25</math>. Since the friend who earned <math>\$40</math> will need to leave with <math>\$25</math>, he will have to give <math>\$\left(40-25\right)=\boxed{\textbf{(C) }\$15}</math> to the others.
 +
 
==Video Solution by WhyMath==
 
==Video Solution by WhyMath==
 
https://youtu.be/-mSgttsOv2Y
 
https://youtu.be/-mSgttsOv2Y

Revision as of 21:53, 10 January 2022

Problem

Four friends do yardwork for their neighbors over the weekend, earning $$15, $20, $25,$ and $$40,$ respectively. They decide to split their earnings equally among themselves. In total how much will the friend who earned $$40$ give to the others?

$\textbf{(A) }$5 \qquad \textbf{(B) }$10 \qquad \textbf{(C) }$15 \qquad \textbf{(D) }$20 \qquad \textbf{(E) }$25$

Solution

The friends earn $$\left(15+20+25+40\right)=$100$ in total. Since they decided to split their earnings equally, it follows that each person will get $$\left(\frac{100}{4}\right)=$25$. Since the friend who earned $$40$ will need to leave with $$25$, he will have to give $$\left(40-25\right)=\boxed{\textbf{(C) }$15}$ to the others.

Video Solution by WhyMath

https://youtu.be/-mSgttsOv2Y

~savannahsolver

Video Solution by North America Math Contest Go Go Go

https://www.youtube.com/watch?v=ZwfPEYd55NQ

~North America Math Contest Go Go Go

Video Solution

https://youtu.be/eSxzI8P9_h8 ~ The Learning Royal

Video Solution by Interstigation

https://youtu.be/YnwkBZTv5Fw?t=62

~Interstigation

See Also

2020 AMC 8 (ProblemsAnswer KeyResources)
Preceded by
Problem 1
Followed by
Problem 3
1 2 3 4 5 6 7 8 9 10 11 12 13 14 15 16 17 18 19 20 21 22 23 24 25
All AJHSME/AMC 8 Problems and Solutions

The problems on this page are copyrighted by the Mathematical Association of America's American Mathematics Competitions. AMC logo.png